Absolute Max/Min of Cube Root (8-t) in [0,8]

  • Thread starter Thread starter scorpa
  • Start date Start date
Click For Summary
To find the absolute maximum and minimum of the function (cube root t)(8-t) on the interval [0,8], the first derivative was calculated to identify critical points. The correct first derivative is f'(t) = (8-t)/(3t^(2/3)) - t^(1/3), which simplifies to find where it equals zero. The critical point at t=0 is acknowledged, but confusion arose regarding the numerator of the derivative. After clarification, the participant realized an error in their exponent, making the problem easier to solve. The discussion highlights the importance of correctly deriving functions to find critical values effectively.
scorpa
Messages
367
Reaction score
1
Hello Everyone,

I'm doing a question on critical values and absolute max and mins and for some stupid reason I can't seem to get one of the questions that was assigned.

Find the absolute max and/or min of the (cube root t )(8-t) in the closed interval [0,8]

First to solve this found the first derivative so that I could find the critical numbers of the equation, which is where the first derivative equals zero.
So when I found the first derivative I got ((1/3)(8-t)-t^(2-3))/(cube root t)

Now I know that one of these critical values is obviously going to be at t=0. But I'm stumped when it comes to finding the critical number from the numerator. I know this is a really stupid question, but I just can't seem to figure it out. Thanks in advance for any advice you can give.
 
Physics news on Phys.org
Your derivative seems to be a bit off.

\mbox{If } f(t) = t^\frac{1}{3}\cdtot (8-t)

f'(t) = \frac{1}{3}\cdot t^\frac{-2}{3}\cdot (8-t) - t^\frac{1}{3} = 0
f'(t) = \frac{(8-t)}{3t^\frac{2}{3}} - \frac{3t}{3t^\frac{2}{3}} = 0
 
Oh geez, I can't believe myself. I wrote the wrong exponent when I was doing the first derivative, I can be such an idiot sometimes. Thanks a lot, the question is a lot easier now...haha.
 
Question: A clock's minute hand has length 4 and its hour hand has length 3. What is the distance between the tips at the moment when it is increasing most rapidly?(Putnam Exam Question) Answer: Making assumption that both the hands moves at constant angular velocities, the answer is ## \sqrt{7} .## But don't you think this assumption is somewhat doubtful and wrong?

Similar threads

Replies
2
Views
2K
  • · Replies 30 ·
2
Replies
30
Views
3K
  • · Replies 5 ·
Replies
5
Views
1K
  • · Replies 4 ·
Replies
4
Views
3K
Replies
1
Views
2K
  • · Replies 2 ·
Replies
2
Views
6K
  • · Replies 5 ·
Replies
5
Views
2K
Replies
1
Views
1K
  • · Replies 4 ·
Replies
4
Views
2K
  • · Replies 1 ·
Replies
1
Views
2K